Você está na página 1de 15

CBSEPracticalSkills.

com

Edulabz International

REAL NUMBERS
Exercise 1.1

Q.1. Use Euclids division algorithm to find the HCF of:


(i) 135 and 225
(ii) 196 and 38220
(iii) 867 and 255
Solution. (i) In 135 and 225, 225 is larger integer.
Using Euclids division algorithm,
[Where 135 is divisor, 90 is remainder]
225 = 135 1 + 90
Since, remainder 90 0 , by applying Eudids division algorithm to 135 and 90

135 = 90 1 + 45
Again since, remainder 45 0 , by applying Eudids division algorithm to 90 and 45

90 = 45 2 + 0
Now, the remainder is zero so, our procedure stops.
Hence, HCF of 135 and 225 is 45.
(ii) In 196 and 38220, 38220 is larger integer.
Using Euclids division algorithm,
38220 = 196 195 + 0
remainder = 0
The remainder is zero so our procedure stops.
Hence, HCF of 196 and 38220 is 196.
(iii) In 867 and 255, 867 is larger integer.
Using Euclids division algorithm
867 = 255 3 + 102 [Where 255 is divisor, 102 is remainder]
Since, remainder 102 0 , by applying Eudids division algorithm to 255 and 102
255 = 102 2 + 51
Since, remainder 51 0 , by applying Eudids division algorithm 102 and 51
So,
102 = 51 2 + 0
remainder = 0
The remainder is zero so our procedure stops.
Hence, HCF of 867 and 255 is 51.

Q.2. Show that any positive odd integer is of the form 6q + 1 or 6q + 3 or 6q + 5,


where q is some integer.

CBSEPracticalSkills.com

Edulabz International

CBSEPracticalSkills.com

Edulabz International

Solution. Let a be any positive odd integer. We apply Euclids division algorithm with
a and b = 6 .
Since 0 r < 6, the positive remainders are 0, 1, 2, 3, 4 and 5.
i.e., a can be 6q or 6q + 1, or 6q + 2 , or 6q + 3 , or 6q + 4 , or 6q + 5 where q is the
quotient. But, a is odd so a cannot be equal to 6q, 6q + 2, 6q + 4 which are even
numbers. (divisible by 2)
Any odd integer is of the form 6q + 1 or 6q + 3 or 6q + 5.
Q.3. An army contingent of 616 members is to march behind an army band of 32
members in a parade. The two groups are to march in the same number of
columns. What is the maximum number of columns in which they can march?
Solution. Total members in army are 616 and 32, 616 is larger integer i.e. 616 > 32
Using Euclids division algorithm,
[Where 32 is divisor, 8 is remainder]
616 = 32 19 + 8
Since, remainder 8 0 , by applying Eudids division algorithm, we get
32 = 8 4 + 0
The remainder is zero,
Hence, the maximum numbers of columns in which both 616 members and
32 members can march is 8 columns.
Q.4. Use Euclids division lemma to show that the square of any positive integer is
either of the form 3m or 3m + 1 for some integer m.
Solution. Let x be any positive integer, then it is of the form 3q, 3q + 1 or 3q + 2 .
So

x = 3q

( x) 2 = (3q) 2 (Squaring both sides)

x 2 = 9q 2 = 3(3q 2 ) = 3m , (where m = 3q 2 )

Hence
If

x 2 = 3m
x = 3q + 1

...(i)

x 2 = (3q + 1) 2 (Squaring both sides)

x 2 = 9q 2 + 1 + 2 3q 1

x 2 = 3(3q 2 + 2q ) + 1

x 2 = 3m + 1

where,

...(ii)

m = 3q 2 + 2q

From equation (i) and (ii), we get


x 2 = 3m, 3m + 1

CBSEPracticalSkills.com

Edulabz International

CBSEPracticalSkills.com

Edulabz International

Square of any positive integer is either of the form 3m or 3m + 1 for some


integer m.
Q.5. Use Euclids division lemma to show that the cube of any positive integer is of
the form 9m, 9m + 1 or 9m + 8.
Solution. Let a be any positive integer, then it is of the form 3q, 3q + 1 or 3q + 2 .

If

a = 3q

then

a 3 = 27q 3 (cubing both sides)

a 3 = 9 3q3 = 9m

...(i)

where m = (3q3 )

a = (3q + 1)

If

a3 = (3q + 1)3 (cubing both sides)

[We know (a + b)3 = a 3 + b3 + 3a 2b + 3b 2 a ]

a3 = (3q)3 + (1)3 + 3 (3q )2 1 + 3 1 3q

a3 = 27q3 + 1 + 27q 2 + 9q

a3 = 27q3 + 27q 2 + 9q + 1

a3 = 9(3q3 + 3q 2 + q) + 1

a 3 = 9m + 1

...(ii)

Where m = 3q3 + 3q 2 + q

a = 3q + 2

If

a 3 = (3q + 2)3 (cubing both sides)

a3 = (3q)3 + (2)3 + 3 (3q ) 2 2 + 3 (2) 2 3q

a3 = 27q3 + 8 + 54q 2 + 36q

a3 = 27q3 + 54q 2 + 36q + 8

a3 = 9(3q3 + 6q 2 + 4q) + 8

a 3 = 9m + 8

...(iii)

Where m = 3q3 + 6q 2 + 4q
Hence, from equation (i), (ii) and (iii) the cube of any integer is of the form
9m, 9m + 1 or 9m + 8.

Exercise 1.2
Q.1. Express each number as a product of its prime factors:

CBSEPracticalSkills.com

Edulabz International

CBSEPracticalSkills.com

Edulabz International

(i) 140
(iii) 3825
(v) 7429

(ii) 156
(iv) 5005

Solution. (i) 140


140
70

35

Hence, 140 = 2 2 5 7 = 22 5 7
(ii) 156
156
78

39

13

Hence, 156 = 2 2 3 13 = 22 3 13
(iii) 3825
3825
3

1275
425

3
5

85
5

17

Hence, 3825 = 3 3 5 5 17 = 32 52 17
(iv) 5005
5005
5

1001
7

143
11

CBSEPracticalSkills.com

13

Edulabz International

CBSEPracticalSkills.com

Edulabz International

Hence, 5005 = 5 7 11 13
(v) 7429
7429
17

437
19

23

Hence, 7429 = 17 19 23
Q.2. Find the LCM and HCF of the following pairs of integers and verify that
LCM HCF = Product of the two numbers.
(i) 26 and 91
(ii) 510 and 92
(iii) 336 and 54
Solution. (i) 26 and 91

26 = 2 13
91 = 7 13
HCF = 13
LCM = 2 7 13 = 182
Verification:
LCM HCF = 182 13 = 2366
Product of two numbers = 26 91 = 2366

Product of two numbers = LCM HCF

(ii) 510 and 92

CBSEPracticalSkills.com

Edulabz International

CBSEPracticalSkills.com

Edulabz International

510 = 2 3 5 17
92 = 2 2 23

HCF = 2

LCM = 2 2 3 5 17 23 = 23460
Verification:
LCM HCF = 2 23460 = 46920
Product of two numbers = 510 92 = 46920

Product of two numbers = LCM HCF

So,

336
2

54
168

2
84

2
2

27
9

3
42

21
7

(iii) 336 and 54


336 = 2 2 2 2 3 7
54 = 2 3 3 3
HCF = 2 3 = 6
LCM = 2 2 2 2 3 3 3 7 = 3024
Verification:
LCM HCF = 6 3024 = 18144
Product of two numbers = 336 54 = 18144

Product of two numbers = LCM HCF


Q.3. Find the LCM and HCF of the following integers by applying the prime
factorisation method.
(i) 12, 15 and 21
(ii) 17, 23 and 29
(iii) 8, 9 and 25
Solution. (i) 12, 15 and 21

CBSEPracticalSkills.com

Edulabz International

CBSEPracticalSkills.com

Edulabz International

Prime factorisation of 12 = 2 2 3
Prime factorisation of 15 = 3 5
Prime factorisation of 21 = 3 7
HCF of 12, 15 and 21 = 3
LCM of 12, 15 and 21 = 2 2 3 5 7 = 420
17
1

23
17

29
23

29

(ii) 17, 23 and 29


Prime factorisation of 17 = 1 17
Prime factorisation of 23 = 1 23
Prime factorisation of 29 = 1 29
HCF of 17, 23 and 29 = 1
LCM of 17, 23 and 29 = 1 17 23 29 = 11339
(iii) 8, 9 and 25
8
2

9
4

25
3

Prime factorisation of 8 = 1 2 2 2
Prime factorisation of 9 = 1 3 3
Prime factorisation of 25 = 1 5 5
HCF of 8, 9 and 25 = 1
LCM of 8, 9 and 25 = 2 2 2 3 3 5 5 = 1800
Q.4. Given that HCF (306, 657) = 9, find LCM (306, 657).
Solution. We know that, HCF LCM = Product of the two numbers
HCF (306, 657) = 9 (given)
9 LCM = 306 657

306 657
= 22338
LCM =

9
Q.5. Check whether 6n can end with the digit 0 for any natural number n.
Solution. Let number 6n end with the digit 0 for any n N . Then, 6n will be divisible
by 5. But, prime factors of 6 are 2 and 3.
Prime factor of (6) n are (2 3) n

So, it is clear that in prime factorisation of 6n there is no 5.

CBSEPracticalSkills.com

Edulabz International

CBSEPracticalSkills.com

Edulabz International

By the uniqueness of the Fundamental Theorem of Arithmetic, every composite


number can be expressed as a product of primes and this factorisation is unique, apart
from the order in which the prime factors occur.
Hence, there exists no any natural number n for which 6n ends with the digit zero.
Q.6. Explain why 7 1113 + 13 and 7 6 5 4 3 2 1 + 5 are composite
numbers.
Solution. 7 1113 + 13 = 13 ( 7 11 + 1) = 13 78 ,

which is not a prime number because it has more than two factors. So, it is a
composite number.
And,

7 6 5 4 3 2 1 + 5 = 5 ( 7 6 4 3 2 + 1) = 5 1009 ,

which is not a prime number because it has more than two factors. So, it is also a
composite number.
Q.7. There is a circular path around a sports field. Sonia takes 18 minutes to drive
one round of the field, while Ravi takes 12 minutes for the same. Suppose they
both start at the same point and at the same time, and go in the same direction.
After how many minutes will they meet again at the starting point?
Solution. Time taken by Sonia to drive one round of the field = 18 minutes.
Time taken by Ravi to drive one round of the field = 12 minutes.
2
2
3
3

18, 12
9, 6
9, 3
3, 1
1, 1

So, LCM of 18 and 12 = 2 2 3 3 = 36


Hence, they will meet again at the starting point after 36 minutes.

Exercise 1.3
5 is irrational.

Q.1. Prove that


Solution. Let

5 is rational.

r
(where, r and s are integers and s 0)
s
Where r and s have no common factor except 1.
Now by squaring both sides, we get
5=

r
( 5) =
s

5=

r2
s2

CBSEPracticalSkills.com

Edulabz International

CBSEPracticalSkills.com

Edulabz International

r 2 = 5s 2

...(i)

5 is a factor of r 2
Since, 5 is a prime number, if 5 divides r2, then, 5 also divides r.
r = 5n, where n is an integer.
Let
r = 5n in equation (i), we get,

Put

(5n) 2 = 5s 2

25n 2 = 5s 2

s 2 = 5n 2

5 is a factor of s 2 . As, 5 is a prime number, 5 also divides s.


So, 5 is a common factor of r and s. But it contracts the facts that r and s has only
one common factor i.e. 1.
Hence,

5 is an irrational number.

Q.2. Prove that 3 + 2 5 is irrational.


Solution. Let 3 + 2 5 is a rational number.

Then we can find coprime numbers a and b.


a
Such that 3 + 2 5 =
b
a
2 5 = 3

b
5=

So,

(where b 0 )

1 a
3 is rational
2 b

5 is rational.

But it contracts the fact that

5 is irrational

Hence, 3 + 2 5 is irrational.
Q.3. Prove that the following are irrationals:
1
(i)
(ii) 7 5
2
(iii) 6 + 2
Solution. (i) Let

So,

1
2

1
a
= , where a and b are co-prime and b 0.
2 b

CBSEPracticalSkills.com

Edulabz International

CBSEPracticalSkills.com

Edulabz International

b
= 2
a

b=a 2

b
is a rational i.e,
a

But it contracts the fact that


1
Hence,
is irrational.
2

2 is rational.
2 is irrational.

(ii) Let 7 5 is a rational.

a
where a and b are co - prime and b 0.
b
a
5=

7b
a
Since a and b are integers,
is rational, and 5 is also rational. But, it contracts
7b
the fact that 5 is irrational.
7 5=

Hence, 7 5 is irrational.
(iii) Let 6 + 2 is rational.
a
6 + 2 = where a and b are co - prime and b 0.
b
a
2 = 6

b
a
a
Since, is rational so, 6 is rational, also 2 is rational. But it is a
b
b
contractation as 2 is irrational.
Hence, 6 + 2 is irrational.

Exercise 1.4
Q.1. Without actually performing the long division, state whether the following
rational numbers will have a terminating decimal expansion or a nonterminating repeating decimal expansion:
13
17
(i)
(ii)
3125
8
64
15
(iii)
(iv)
455
1600
29
23
(v)
(vi) 3 2
343
2 5

CBSEPracticalSkills.com

10

Edulabz International

CBSEPracticalSkills.com
(vii)

129

2 2 57 7 5
35
(ix)
50

Solution. (i) Let x =

13
13
= 5
3125 5

Comparing equation (1) with x =

Edulabz International

(viii)

6
15

(x)

77
210

...(1)

p
, we get p = 13 and q = 55
q

Here q is of the form 2n 5m (n = 0, m = 5)

13
have a terminating decimal expansion.
3125
17 17
...(i)
(ii) Let x =
=
8 23
p
Comparing equation (2) with x = , we get p = 17 and q = 23
q
Thus, x =

Here q is of the form 2n 5m (n = 3, m = 0)


Thus, x =

(iii) Let x =

17
have a terminating decimal expansion.
8

64
64
=
455 5 7 13

Comparing equation (3) with x =

...(3)
p
, we get p = 64 and q = 5 7 13
q

Here q is not of the form 2n 5m


64
has a non-terminating repeating decimal expansion.
Thus, x =
455
15
15
= 6 2
(iv) Let x =
...(4)
1600 2 5
Comparing equation (4) with x =

p
, we get p = 15 and q = 26 52
q

Here q is of the form 2n 5m (n = 6, m = 2)

15
having a terminating decimal expansion.
1600
29 29
(v) Let x =
...(5)
=
343 73
Thus, x =

CBSEPracticalSkills.com

11

Edulabz International

CBSEPracticalSkills.com
Comparing equation (5) with x =

Edulabz International

p
, we get p = 29 and q = 73
q

Here q is not of the form 2n 5m


29
having a non-terminating decimal expansion.
Thus, x =
343
23
(vi) Let x = 3 2
...(6)
25
p
Comparing equation (6) with x = , we get p = 23 and q = 2352
q
Here q is of the form 2n 5m (n = 3, m = 2)
Thus, x =
(vii) Let x =

23

2352
129

have a terminating decimal expansion.

2257 75

Comparing equation (7) with x =

...(7)
p
, we get p = 129 and q = 2557 75
q

Here q is not of the form 2n 5m


129
Thus, x = 2 7 5 have a non-terminating decimal expansion.
2 5 7
6 2
(viii) Let x = =
...(8)
15 5
p
Comparing equation (8) with x = , we get p = 2 and q = 5 = 20 51
q
Here q is of the form 2n 5m (n = 0, m = 1)

6
have a terminating decimal expansion.
15
35 7
=
(ix) Let x =
...(9)
50 10
p
Comparing equation (9) with x = , we get p = 7 and q = 10 = 21 51
q
Thus, x =

Here q is of the form 2n 5m (n = 1, m = 1)

7
have a terminating decimal expansion.
10
77 11
=
...(10)
(x) Let x =
210 30
Thus, x =

CBSEPracticalSkills.com

12

Edulabz International

CBSEPracticalSkills.com

Edulabz International

Comparing equation (10) with x =

p
, we get p = 11 and q = 30 = 21 31 51
q

Here q is not of the form 2n 5m


77
have a non-terminating decimal expansion.
Thus, x =
210
Q.2. Write down the decimal expansions of those rational numbers in Question 1
above which have terminating decimal expansions.
Solution. From question 1, above, we know that only (i), (ii), (iv), (vi), (viii) and (ix)
have terminating decimal expansions.
x=

(i)

13
13
= 5
3125 5

13 25

x=

x=

x=

13
= x = 0.00416
3125
17 17
x=
=
8 23

(ii)

(Multiply and divide by 25 )

5 2
13 32
5

(2 5)5

416
(10)5

416
100000

17 53

x=

x=

x=

x = 2.125
17
= x = 2.125
8
15
x=
1600
15
(Multiply and divide by 54 )
x= 6 2
2 5

(iv)

(Multiply and divide with 53 )

2 5
17 125
3

(2 5)3
2125
(10)

2125
1000

CBSEPracticalSkills.com

13

Edulabz International

CBSEPracticalSkills.com
15 54

x=

x=

x=

x=

So,

15
= x = 0.009375
1600

(vi)

So,
(viii)

So,
(ix)

So,

Edulabz International

26 52 54
15 625

x=

26 56
9375

(2 5)6
9375
(10)

23

9375
= 0.009375
1000000

23 5

115

2352 23 52 5 23 53
115
115
x=
=
= 0.115
3
1000
(2 5)
23

= x = 0.115
2352
6 2
x= =
15 5

x=

2 21
2 5
1

4
= 0.4
10

6
= x = 0.4
15
35 7
7
x=
=
= 1 1
50 10 2 5
7
7
x=
=
= 0.7
1
(2 5)
(10)1
35
= x = 0.7
50

Q.3. The following real numbers have decimal expansions as given below. In each
case, decide whether they are rational or not. If they are rational, and of the
p
form , what can you say about the prime factors of q?
q
(i) 43.123456789
(ii) 0.120120012000120000...
(iii) 43.123456789

CBSEPracticalSkills.com

14

Edulabz International

CBSEPracticalSkills.com

Edulabz International

Solution. (i) Let x = 43.123456789


...(i)
It is clear from the number that x is a rational number.
43123456789
x=
1000000000
43123456789
...(ii)

x=
109

From equation (ii) we see that x is a rational number and of the form
where

p
.
q

p = 43123456789 and q = 109 = 25 59

Prime factors of q are 29 59

(ii) Let x = 0.120120012000120000...


Since x neither terminating nor non-terminating repeating. So, is it is an irrational
number.
(iii) Let x = 43.123456789
...(1)
It is clear that the given number is a rational number because it is a
non-terminating and repeating decimal.
Multiply equation (i) with 109 on both sides, we get
1000000000x = 43123456789.123456289
(2)
Subtracting (i) from (2), we get
1000000000 x = 43123456789.123456789...
x = 43.123456789...

999999999 x = 43123456746

x=

43123456746
999999999

Which is a rational number of the form

x=

x=

p
.
q

4791495194
, where p = 4791495194 and q = 111111111
111111111
4791495194
32 (12345679)

Which is not of the form 2n 5m , n, m I .

CBSEPracticalSkills.com

15

Edulabz International

Você também pode gostar